[Resolução] 10.4.18

Disciplina de Cálculo IV do ICMC
Locked
Eugenio Massa
Posts: 2
Joined: 31 Aug 2022 11:12
Been thanked: 1 time

[Resolução] 10.4.18

Post by Eugenio Massa »

Enunciado:
Determinar se há convergência na série \(f(n) = 1 + \dfrac{n}{n+1} \cdot \cos{\dfrac{n \pi}{2}}\). Em caso positivo, indicar para qual valor converge.
Resposta: a série \(\{f(n)\}\) diverge.

Resolução:

Suponhamos que o contrário seja verdade, ou seja, que exista um número real \(L\) e um número inteiro positivo \(N\) tais que

\(\bigg|f(n) - L\bigg| < \varepsilon \quad \forall \; \varepsilon > 0 \quad \forall \; n > N\).

Como \(N\) é positivo, \(4N > N\) e \(4N + 2 > N\).
Com \(\varepsilon = \frac{1}{2}\), temos

\(\bigg|f(4N) - L\bigg| < \dfrac{1}{2} \quad \implies \quad \bigg|1 + \bigg(\dfrac{4N}{4N+1}\bigg)\cos{(2N \pi)} - L\bigg| < \dfrac{1}{2}\)

e

\(\bigg|f(4N + 2) - L\bigg| < \dfrac{1}{2} \quad \implies \quad \bigg|1 + \bigg(\dfrac{4N + 2}{4N+3}\bigg)\cos{(2N \pi + \pi)} - L\bigg| < \dfrac{1}{2}\).

A partir disso, temos

\(\bigg|1 + \dfrac{4N}{4N + 1} - L\bigg| < \dfrac{1}{2}\quad\) e \(\quad\bigg|L - 1 + \dfrac{4N + 2}{4N + 3}\bigg| < \dfrac{1}{2}\)

Por fim, podemos somar as inequações:

\(\bigg|1 + \dfrac{4N}{4N + 1} - L\bigg|\quad + \quad\bigg|L - 1 + \dfrac{4N + 2}{4N + 3}\bigg| < 1\\\)
\(\implies \quad \bigg|\dfrac{4N}{4N + 1} + \dfrac{4N + 2}{4N + 3}\bigg| < 1\\\)
\(\implies \quad \dfrac{4N}{4N + 1} + \dfrac{4N + 2}{4N + 3} < 1.\)

Contudo, há uma contradição nessa inequação, uma vez que tanto \(\frac{4N}{4N + 1}\) quanto \(\frac{4N + 2}{4N + 3}\) são maiores que \(\frac{3}{4}\) para todos os valores inteiros positivos de \(N\). Dessa maneira, não existe o limite \(L\) e a série diverge.
Locked